Akademisyenler öncülüğünde matematik/fizik/bilgisayar bilimleri soru cevap platformu

Answers posted by DoganDonmez

563
answers
161
best answers
0 votes
cevaplandı 4 Kasım 2015
$\left(\sqrt{\frac ab}-\sqrt{\frac ba}\right)^2\geq0$ ı açıp düzenle
1 vote
cevaplandı 31 Ekim 2015
Hipotenüsü çap kabul eden bir çember çizelim, uç noktalarına $A,B$ merkeze $O$ diyelim. Tales in
0 votes
cevaplandı 31 Ekim 2015
$$y=x^3,\ \left|\frac{dx}x\right|\leq0,01$$ $$dy=3x^2\,dx,\ \left|\frac{dy}y\right|=3\left|\frac{
0 votes
cevaplandı 30 Ekim 2015
$x$, 1. kişinin, $y$ 2. kişinin varış zamanının dakikası olsun.  $0\leq x,y\leq 60 $ olur. Tüm ol
0 votes
0 votes
cevaplandı 20 Ekim 2015
Bazan, bir bölüm (denklik sınıflarının) kümesinden, başka bir kümeye bir $f:A/\sim\to B$ fonksiyo
0 votes
cevaplandı 17 Ekim 2015
Bu soruyu matematiksel çözmek elbette mümkün, bir çözüm yazan çıkar elbet ama ben fiziksel bir çöz
2 votes
cevaplandı 16 Ekim 2015
 de gördüm. orijinal örnek  https://en.wikipedia.org/wiki/Maxima_and_minima#Functions
0 votes
cevaplandı 14 Ekim 2015
Resimle soru sorulduğunda o soruyu, siteden arayıp bulmak (çoğu zaman başlık ve anahtar kelimeler
0 votes
cevaplandı 14 Ekim 2015
LaTeX kodunu herkes görebiliyor sanırım. Formülün üzerine gelip farenin sağ tuşuna basınca "S
1 vote
cevaplandı 10 Ekim 2015
$a_n=(-1)^n$ dizisini gözönüne alalım.. $\lim (a_n)$ yoktur ama, $\limsup (a_n)=1$ dir.
1 vote
cevaplandı 6 Ekim 2015
$[0,1]$ in açık aralıklarla bir örtüsünü alalım. Kompaktlıktan (tıkızlıktan)
1 vote
cevaplandı 3 Ekim 2015
Burada sorun şu: sol tarafta $f$ nin sadece rasyonel sayılardaki değerleri var. Yani fonksiyonun
3 votes
cevaplandı 3 Ekim 2015
Bu soruda, yorumda da belirttiğim gibi, eksik bir varsayım var. $z^i$  nin sonsuz tane değeri var
1 vote
cevaplandı 27 Eylül 2015
Bu soruyu, cevabının çember olduğunu varsaymadan, çözmeliyiz.  Şu iki gerçek soruyu çözüyor:
0 votes
cevaplandı 23 Eylül 2015
Evlere 1,2,3 çeşmelere A,B,C diyelim. Çizilebildiğini sayıp çelişki elde edeceğiz. Bu çelişkiyi b
1 vote
cevaplandı 15 Eylül 2015
Yasin Şale bu sorunun standart Analiz çözümünü gayet güzel yazmış Başka geometrik bir çözü
3 votes
cevaplandı 12 Eylül 2015
$(a_n)$ artan ve üstten sınırlı bir dizi olsun. $A=\{a_n: n\in\mathbb{N} \}$ olsun ($A$; boş
20,217 soru
21,750 cevap
73,348 yorum
1,972,791 kullanıcı